An inequality for convex function from $mathbb R$ to $mathbb R$












1












$begingroup$


If $f(x): mathbb R to mathbb R$ is a convex function, prove that
$$f(x_1) + f(x_2) + f(x_3) + 3 f(frac{x_1 + x_2 + x_3}{3}) geq 2 f(frac{x_1 + x_2}{2}) + 2 f(frac{x_1 + x_3}{2}) + 2 f(frac{x_2 + x_3}{2})$$



What I have tried:
If two of $x_1$, $x_2$, $x_3$ are equal, without loss of generality, assuming $x_2 = x_3$, then the inequality degenerate to
begin{align}
f(x_1) + 3 f(frac{x_1 + 2 x_2}{3}) geq 4 f(frac{x_1 + x_2}{2})
end{align}

Since $f$ is convex, we have
$
frac{1}{4} f(x_1) + frac{3}{4} f(frac{x_1 + 2 x_2}{3})
geq f(frac{1}{4} x_1 + frac{3}{4} frac{x_1 + 2 x_2}{3})
geq f(frac{x_1 + x_2}{2})
$
, thus the inequality holds.



Then we consider the case where $x_1$, $x_2$, $x_3$ are distinct. Without loss of generality, we assume $0 = x_1 < x_2 = lambda leq frac{1}{2} < x_3 = 1$. Then the inequality degenerates to
begin{align}
frac{1}{2} f(0) + frac{1}{2} f(lambda) + frac{3}{2} f(frac{lambda + 1}{3}) + frac{1}{2} f(1) geq f(frac{lambda}{2}) + f(frac{1}{2}) + f(frac{lambda + 1}{2})
end{align}

If $x_2 = frac{1}{2}$, it further degenerates to
$frac{1}{2} f(0) + frac{1}{2} f(0.5) + frac{1}{2} f(0.5) + frac{1}{2} f(1) geq f(0.25) + f(0.75),$
which is obviously true. Thus we can only consider $0 < lambda < frac{1}{2}$ (then $0 < frac{lambda}{2} < lambda < frac{lambda + 1}{3} < frac{1}{2} < frac{lambda + 1}{2} < 1$).










share|cite|improve this question









$endgroup$








  • 1




    $begingroup$
    This is known as Popoviciu's inequality
    $endgroup$
    – munchhausen
    Dec 19 '18 at 14:49
















1












$begingroup$


If $f(x): mathbb R to mathbb R$ is a convex function, prove that
$$f(x_1) + f(x_2) + f(x_3) + 3 f(frac{x_1 + x_2 + x_3}{3}) geq 2 f(frac{x_1 + x_2}{2}) + 2 f(frac{x_1 + x_3}{2}) + 2 f(frac{x_2 + x_3}{2})$$



What I have tried:
If two of $x_1$, $x_2$, $x_3$ are equal, without loss of generality, assuming $x_2 = x_3$, then the inequality degenerate to
begin{align}
f(x_1) + 3 f(frac{x_1 + 2 x_2}{3}) geq 4 f(frac{x_1 + x_2}{2})
end{align}

Since $f$ is convex, we have
$
frac{1}{4} f(x_1) + frac{3}{4} f(frac{x_1 + 2 x_2}{3})
geq f(frac{1}{4} x_1 + frac{3}{4} frac{x_1 + 2 x_2}{3})
geq f(frac{x_1 + x_2}{2})
$
, thus the inequality holds.



Then we consider the case where $x_1$, $x_2$, $x_3$ are distinct. Without loss of generality, we assume $0 = x_1 < x_2 = lambda leq frac{1}{2} < x_3 = 1$. Then the inequality degenerates to
begin{align}
frac{1}{2} f(0) + frac{1}{2} f(lambda) + frac{3}{2} f(frac{lambda + 1}{3}) + frac{1}{2} f(1) geq f(frac{lambda}{2}) + f(frac{1}{2}) + f(frac{lambda + 1}{2})
end{align}

If $x_2 = frac{1}{2}$, it further degenerates to
$frac{1}{2} f(0) + frac{1}{2} f(0.5) + frac{1}{2} f(0.5) + frac{1}{2} f(1) geq f(0.25) + f(0.75),$
which is obviously true. Thus we can only consider $0 < lambda < frac{1}{2}$ (then $0 < frac{lambda}{2} < lambda < frac{lambda + 1}{3} < frac{1}{2} < frac{lambda + 1}{2} < 1$).










share|cite|improve this question









$endgroup$








  • 1




    $begingroup$
    This is known as Popoviciu's inequality
    $endgroup$
    – munchhausen
    Dec 19 '18 at 14:49














1












1








1


1



$begingroup$


If $f(x): mathbb R to mathbb R$ is a convex function, prove that
$$f(x_1) + f(x_2) + f(x_3) + 3 f(frac{x_1 + x_2 + x_3}{3}) geq 2 f(frac{x_1 + x_2}{2}) + 2 f(frac{x_1 + x_3}{2}) + 2 f(frac{x_2 + x_3}{2})$$



What I have tried:
If two of $x_1$, $x_2$, $x_3$ are equal, without loss of generality, assuming $x_2 = x_3$, then the inequality degenerate to
begin{align}
f(x_1) + 3 f(frac{x_1 + 2 x_2}{3}) geq 4 f(frac{x_1 + x_2}{2})
end{align}

Since $f$ is convex, we have
$
frac{1}{4} f(x_1) + frac{3}{4} f(frac{x_1 + 2 x_2}{3})
geq f(frac{1}{4} x_1 + frac{3}{4} frac{x_1 + 2 x_2}{3})
geq f(frac{x_1 + x_2}{2})
$
, thus the inequality holds.



Then we consider the case where $x_1$, $x_2$, $x_3$ are distinct. Without loss of generality, we assume $0 = x_1 < x_2 = lambda leq frac{1}{2} < x_3 = 1$. Then the inequality degenerates to
begin{align}
frac{1}{2} f(0) + frac{1}{2} f(lambda) + frac{3}{2} f(frac{lambda + 1}{3}) + frac{1}{2} f(1) geq f(frac{lambda}{2}) + f(frac{1}{2}) + f(frac{lambda + 1}{2})
end{align}

If $x_2 = frac{1}{2}$, it further degenerates to
$frac{1}{2} f(0) + frac{1}{2} f(0.5) + frac{1}{2} f(0.5) + frac{1}{2} f(1) geq f(0.25) + f(0.75),$
which is obviously true. Thus we can only consider $0 < lambda < frac{1}{2}$ (then $0 < frac{lambda}{2} < lambda < frac{lambda + 1}{3} < frac{1}{2} < frac{lambda + 1}{2} < 1$).










share|cite|improve this question









$endgroup$




If $f(x): mathbb R to mathbb R$ is a convex function, prove that
$$f(x_1) + f(x_2) + f(x_3) + 3 f(frac{x_1 + x_2 + x_3}{3}) geq 2 f(frac{x_1 + x_2}{2}) + 2 f(frac{x_1 + x_3}{2}) + 2 f(frac{x_2 + x_3}{2})$$



What I have tried:
If two of $x_1$, $x_2$, $x_3$ are equal, without loss of generality, assuming $x_2 = x_3$, then the inequality degenerate to
begin{align}
f(x_1) + 3 f(frac{x_1 + 2 x_2}{3}) geq 4 f(frac{x_1 + x_2}{2})
end{align}

Since $f$ is convex, we have
$
frac{1}{4} f(x_1) + frac{3}{4} f(frac{x_1 + 2 x_2}{3})
geq f(frac{1}{4} x_1 + frac{3}{4} frac{x_1 + 2 x_2}{3})
geq f(frac{x_1 + x_2}{2})
$
, thus the inequality holds.



Then we consider the case where $x_1$, $x_2$, $x_3$ are distinct. Without loss of generality, we assume $0 = x_1 < x_2 = lambda leq frac{1}{2} < x_3 = 1$. Then the inequality degenerates to
begin{align}
frac{1}{2} f(0) + frac{1}{2} f(lambda) + frac{3}{2} f(frac{lambda + 1}{3}) + frac{1}{2} f(1) geq f(frac{lambda}{2}) + f(frac{1}{2}) + f(frac{lambda + 1}{2})
end{align}

If $x_2 = frac{1}{2}$, it further degenerates to
$frac{1}{2} f(0) + frac{1}{2} f(0.5) + frac{1}{2} f(0.5) + frac{1}{2} f(1) geq f(0.25) + f(0.75),$
which is obviously true. Thus we can only consider $0 < lambda < frac{1}{2}$ (then $0 < frac{lambda}{2} < lambda < frac{lambda + 1}{3} < frac{1}{2} < frac{lambda + 1}{2} < 1$).







convex-analysis convex-optimization jensen-inequality






share|cite|improve this question













share|cite|improve this question











share|cite|improve this question




share|cite|improve this question










asked Dec 19 '18 at 14:13









Rubisco LeeRubisco Lee

1148




1148








  • 1




    $begingroup$
    This is known as Popoviciu's inequality
    $endgroup$
    – munchhausen
    Dec 19 '18 at 14:49














  • 1




    $begingroup$
    This is known as Popoviciu's inequality
    $endgroup$
    – munchhausen
    Dec 19 '18 at 14:49








1




1




$begingroup$
This is known as Popoviciu's inequality
$endgroup$
– munchhausen
Dec 19 '18 at 14:49




$begingroup$
This is known as Popoviciu's inequality
$endgroup$
– munchhausen
Dec 19 '18 at 14:49










1 Answer
1






active

oldest

votes


















3












$begingroup$

A proof of Popoviciu's inequality follows from Jensen's Inequality for convex functions. Wlog, assume $x le y le z$ there are two possibilities. Either $displaystyle y < frac{x+y+z}{3}$ or the reverse.



In the first case, $displaystyle frac{x+y+z}{3} < frac{x+z}{2}< z$ and $displaystyle frac{x+y+z}{3} < frac{y+z}{2} < z$, thus, there are positive real numbers $lambda_1,lambda_2 >0$ such that,



$displaystyle frac{x+z}{2} = lambda_1 z + (1-lambda_1)left(frac{x+y+z}{3}right)$ and $displaystyle frac{y+z}{2} = lambda_2 z + (1-lambda_2)left(frac{x+y+z}{3}right)$



It is easy to verify that $lambda_1 + lambda_2 = dfrac{1}{2}$.



Now, using Jensen's inequality one has:



$displaystyle fleft(frac{x+y}{2}right) le frac{f(x)+f(y)}{2}$



$displaystyle fleft(frac{x+z}{2}right) = lambda_1 f(z) + (1-lambda_1)fleft(frac{x+y+z}{3}right)$



$displaystyle fleft(frac{y+z}{2}right) = lambda_2 f(z) + (1-lambda_2)fleft(frac{x+y+z}{3}right)$



Adding the three inequalities gives the Popoviciu's inequality. The second case where $displaystyle y > frac{x+y+z}{3}$ we simply interchange the roles of $x$ and $z$ and the rest is similar.






share|cite|improve this answer









$endgroup$













    Your Answer





    StackExchange.ifUsing("editor", function () {
    return StackExchange.using("mathjaxEditing", function () {
    StackExchange.MarkdownEditor.creationCallbacks.add(function (editor, postfix) {
    StackExchange.mathjaxEditing.prepareWmdForMathJax(editor, postfix, [["$", "$"], ["\\(","\\)"]]);
    });
    });
    }, "mathjax-editing");

    StackExchange.ready(function() {
    var channelOptions = {
    tags: "".split(" "),
    id: "69"
    };
    initTagRenderer("".split(" "), "".split(" "), channelOptions);

    StackExchange.using("externalEditor", function() {
    // Have to fire editor after snippets, if snippets enabled
    if (StackExchange.settings.snippets.snippetsEnabled) {
    StackExchange.using("snippets", function() {
    createEditor();
    });
    }
    else {
    createEditor();
    }
    });

    function createEditor() {
    StackExchange.prepareEditor({
    heartbeatType: 'answer',
    autoActivateHeartbeat: false,
    convertImagesToLinks: true,
    noModals: true,
    showLowRepImageUploadWarning: true,
    reputationToPostImages: 10,
    bindNavPrevention: true,
    postfix: "",
    imageUploader: {
    brandingHtml: "Powered by u003ca class="icon-imgur-white" href="https://imgur.com/"u003eu003c/au003e",
    contentPolicyHtml: "User contributions licensed under u003ca href="https://creativecommons.org/licenses/by-sa/3.0/"u003ecc by-sa 3.0 with attribution requiredu003c/au003e u003ca href="https://stackoverflow.com/legal/content-policy"u003e(content policy)u003c/au003e",
    allowUrls: true
    },
    noCode: true, onDemand: true,
    discardSelector: ".discard-answer"
    ,immediatelyShowMarkdownHelp:true
    });


    }
    });














    draft saved

    draft discarded


















    StackExchange.ready(
    function () {
    StackExchange.openid.initPostLogin('.new-post-login', 'https%3a%2f%2fmath.stackexchange.com%2fquestions%2f3046440%2fan-inequality-for-convex-function-from-mathbb-r-to-mathbb-r%23new-answer', 'question_page');
    }
    );

    Post as a guest















    Required, but never shown

























    1 Answer
    1






    active

    oldest

    votes








    1 Answer
    1






    active

    oldest

    votes









    active

    oldest

    votes






    active

    oldest

    votes









    3












    $begingroup$

    A proof of Popoviciu's inequality follows from Jensen's Inequality for convex functions. Wlog, assume $x le y le z$ there are two possibilities. Either $displaystyle y < frac{x+y+z}{3}$ or the reverse.



    In the first case, $displaystyle frac{x+y+z}{3} < frac{x+z}{2}< z$ and $displaystyle frac{x+y+z}{3} < frac{y+z}{2} < z$, thus, there are positive real numbers $lambda_1,lambda_2 >0$ such that,



    $displaystyle frac{x+z}{2} = lambda_1 z + (1-lambda_1)left(frac{x+y+z}{3}right)$ and $displaystyle frac{y+z}{2} = lambda_2 z + (1-lambda_2)left(frac{x+y+z}{3}right)$



    It is easy to verify that $lambda_1 + lambda_2 = dfrac{1}{2}$.



    Now, using Jensen's inequality one has:



    $displaystyle fleft(frac{x+y}{2}right) le frac{f(x)+f(y)}{2}$



    $displaystyle fleft(frac{x+z}{2}right) = lambda_1 f(z) + (1-lambda_1)fleft(frac{x+y+z}{3}right)$



    $displaystyle fleft(frac{y+z}{2}right) = lambda_2 f(z) + (1-lambda_2)fleft(frac{x+y+z}{3}right)$



    Adding the three inequalities gives the Popoviciu's inequality. The second case where $displaystyle y > frac{x+y+z}{3}$ we simply interchange the roles of $x$ and $z$ and the rest is similar.






    share|cite|improve this answer









    $endgroup$


















      3












      $begingroup$

      A proof of Popoviciu's inequality follows from Jensen's Inequality for convex functions. Wlog, assume $x le y le z$ there are two possibilities. Either $displaystyle y < frac{x+y+z}{3}$ or the reverse.



      In the first case, $displaystyle frac{x+y+z}{3} < frac{x+z}{2}< z$ and $displaystyle frac{x+y+z}{3} < frac{y+z}{2} < z$, thus, there are positive real numbers $lambda_1,lambda_2 >0$ such that,



      $displaystyle frac{x+z}{2} = lambda_1 z + (1-lambda_1)left(frac{x+y+z}{3}right)$ and $displaystyle frac{y+z}{2} = lambda_2 z + (1-lambda_2)left(frac{x+y+z}{3}right)$



      It is easy to verify that $lambda_1 + lambda_2 = dfrac{1}{2}$.



      Now, using Jensen's inequality one has:



      $displaystyle fleft(frac{x+y}{2}right) le frac{f(x)+f(y)}{2}$



      $displaystyle fleft(frac{x+z}{2}right) = lambda_1 f(z) + (1-lambda_1)fleft(frac{x+y+z}{3}right)$



      $displaystyle fleft(frac{y+z}{2}right) = lambda_2 f(z) + (1-lambda_2)fleft(frac{x+y+z}{3}right)$



      Adding the three inequalities gives the Popoviciu's inequality. The second case where $displaystyle y > frac{x+y+z}{3}$ we simply interchange the roles of $x$ and $z$ and the rest is similar.






      share|cite|improve this answer









      $endgroup$
















        3












        3








        3





        $begingroup$

        A proof of Popoviciu's inequality follows from Jensen's Inequality for convex functions. Wlog, assume $x le y le z$ there are two possibilities. Either $displaystyle y < frac{x+y+z}{3}$ or the reverse.



        In the first case, $displaystyle frac{x+y+z}{3} < frac{x+z}{2}< z$ and $displaystyle frac{x+y+z}{3} < frac{y+z}{2} < z$, thus, there are positive real numbers $lambda_1,lambda_2 >0$ such that,



        $displaystyle frac{x+z}{2} = lambda_1 z + (1-lambda_1)left(frac{x+y+z}{3}right)$ and $displaystyle frac{y+z}{2} = lambda_2 z + (1-lambda_2)left(frac{x+y+z}{3}right)$



        It is easy to verify that $lambda_1 + lambda_2 = dfrac{1}{2}$.



        Now, using Jensen's inequality one has:



        $displaystyle fleft(frac{x+y}{2}right) le frac{f(x)+f(y)}{2}$



        $displaystyle fleft(frac{x+z}{2}right) = lambda_1 f(z) + (1-lambda_1)fleft(frac{x+y+z}{3}right)$



        $displaystyle fleft(frac{y+z}{2}right) = lambda_2 f(z) + (1-lambda_2)fleft(frac{x+y+z}{3}right)$



        Adding the three inequalities gives the Popoviciu's inequality. The second case where $displaystyle y > frac{x+y+z}{3}$ we simply interchange the roles of $x$ and $z$ and the rest is similar.






        share|cite|improve this answer









        $endgroup$



        A proof of Popoviciu's inequality follows from Jensen's Inequality for convex functions. Wlog, assume $x le y le z$ there are two possibilities. Either $displaystyle y < frac{x+y+z}{3}$ or the reverse.



        In the first case, $displaystyle frac{x+y+z}{3} < frac{x+z}{2}< z$ and $displaystyle frac{x+y+z}{3} < frac{y+z}{2} < z$, thus, there are positive real numbers $lambda_1,lambda_2 >0$ such that,



        $displaystyle frac{x+z}{2} = lambda_1 z + (1-lambda_1)left(frac{x+y+z}{3}right)$ and $displaystyle frac{y+z}{2} = lambda_2 z + (1-lambda_2)left(frac{x+y+z}{3}right)$



        It is easy to verify that $lambda_1 + lambda_2 = dfrac{1}{2}$.



        Now, using Jensen's inequality one has:



        $displaystyle fleft(frac{x+y}{2}right) le frac{f(x)+f(y)}{2}$



        $displaystyle fleft(frac{x+z}{2}right) = lambda_1 f(z) + (1-lambda_1)fleft(frac{x+y+z}{3}right)$



        $displaystyle fleft(frac{y+z}{2}right) = lambda_2 f(z) + (1-lambda_2)fleft(frac{x+y+z}{3}right)$



        Adding the three inequalities gives the Popoviciu's inequality. The second case where $displaystyle y > frac{x+y+z}{3}$ we simply interchange the roles of $x$ and $z$ and the rest is similar.







        share|cite|improve this answer












        share|cite|improve this answer



        share|cite|improve this answer










        answered Dec 19 '18 at 14:41









        r9mr9m

        13.3k24171




        13.3k24171






























            draft saved

            draft discarded




















































            Thanks for contributing an answer to Mathematics Stack Exchange!


            • Please be sure to answer the question. Provide details and share your research!

            But avoid



            • Asking for help, clarification, or responding to other answers.

            • Making statements based on opinion; back them up with references or personal experience.


            Use MathJax to format equations. MathJax reference.


            To learn more, see our tips on writing great answers.




            draft saved


            draft discarded














            StackExchange.ready(
            function () {
            StackExchange.openid.initPostLogin('.new-post-login', 'https%3a%2f%2fmath.stackexchange.com%2fquestions%2f3046440%2fan-inequality-for-convex-function-from-mathbb-r-to-mathbb-r%23new-answer', 'question_page');
            }
            );

            Post as a guest















            Required, but never shown





















































            Required, but never shown














            Required, but never shown












            Required, but never shown







            Required, but never shown

































            Required, but never shown














            Required, but never shown












            Required, but never shown







            Required, but never shown







            Popular posts from this blog

            Le Mesnil-Réaume

            Ida-Boy-Ed-Garten

            web3.py web3.isConnected() returns false always